Verbal questions from any Manhattan Prep GMAT Computer Adaptive Test. Topic subject should be the first few words of your question.
Leon
 
 

CR - The anticipated retirement of tens of thousands...

by Leon Sat Jun 14, 2008 4:54 pm

The anticipated retirement of tens of thousands of baby boomers will create an unprecedented opportunity to move significant numbers of people into career-track jobs at family-supporting incomes. Major industries, from health care and construction to automotive repair, will soon face deep shortages of workers as a result of projected growth and boomer retirements. Fortunately, many of these jobs have relatively low barriers to entry and could be filled by out-of-work young people. To achieve this result, the city government should convene employers and educators to determine how best to create paths of upward mobility in these fields.

Which of the following, if true, most weakens the argument?

(A) Immigration reform will limit the pool of available workers.
(B) Government efforts have been shown to affect employment trends only rarely.
(C) The best available positions require skills not possessed by the vast majority of the unemployed.
(D) A small proportion of baby boomers will not retire as soon as is anticipated.
(E) Many out-of-work young people are unaware of these looming employment opportunities.

I narrowed my choices down to (B) and (E).

And then, I chose (E), which is wrong.

I have two questions:

1. In agreeing with the answer key that the correct answer is (B), I would have had to assume that "government efforts" referred to the past efforts of the city government in question. Should I? I have seen other CR questions in which this kind of "government efforts" reference implied efforts by governments in general - city, state, or federal government - instead of the specific one in question, and this kind of generalizing answer was wrong because other government efforts are "irrelevant." Am I reading into this answer choice too much, or could this answer choice be phrased better?

2. I have somewhat of a hard time with the explanation for (E): "If anything, this choice strengthens the argument. If these people are unaware of these opportunities, it would be positive to convene to plan how to reach them." When I think of educators, I think of teachers making a difference in the classrooms. These out-of-work young people are presumably out of school, so I did not make a connection between the educators and the out-of-work young people. And, because of the lack of connection, I chose (E), thinking that no matter how hard educators try to come up with the right educational plans, these plans would be meaningless if they are not taught to people. Is there something wrong in my logic?

Thanks for your help,

Leon
Guest
 
 

by Guest Sun Jun 15, 2008 12:59 am

Which of the following, if true, most weakens the argument?

(A) Immigration reform will limit the pool of available workers.
(B) Government efforts have been shown to affect employment trends only rarely.
(C) The best available positions require skills not possessed by the vast majority of the unemployed.
(D) A small proportion of baby boomers will not retire as soon as is anticipated.
(E) Many out-of-work young people are unaware of these looming employment opportunities.

P: Anticipated boomer retirement + projected growth ---> Woker shortage
P: Young people to fill up the shortage due to low entry barrier
C: City govt. should plan strategy to maintain this upward mobility.

(A) another reason of shortage that further strengthens city govt. plan.
(B) City govt. plan may fail! Because govt. efforts affect trend only rarely.
(C) That is why best plan is necessary. Strengthens.
(D) Not strongly weakens. large proportion of baby boomers may weaken the argument.
(E) That is why city plan is required.

B
Leon
 
 

CR - The anticipated retirement of tens of thousands...

by Leon Sun Jun 15, 2008 4:41 pm

B,

Thanks for your help, but I still need to know where my logic went run. Since you identified (E) as a strengthening answer, you obviously did not reason the same way as I did. Would you mind letting me know where my line of reasoning went wrong?

Thanks,

Leon
Leon
 
 

CR - The anticipated retirement of tens of thousands...

by Leon Sun Jun 15, 2008 4:44 pm

Went "wrong" (not "run").

Staring at the window into the sunny sky is making me want to do something else other than the GMAT...
Maverick
 
 

by Maverick Mon Jun 16, 2008 11:14 am

E) Many out-of-work young people are unaware of these looming employment opportunities.

Government is devising the best strategy to fill up employment opportunities that will arise in future. Now if we know that "Many out-of-work young people are unaware of these looming employment opportunities", will the strategy fail? No. It is not telling us whether the strategy will fail or be successful. It just implies that any strategy must address this unawareness of job-seekers to be successful. The strategy must educate these people about these prospects. But will this effort be successful? We do not no.

But (B) says that despite government efforts there is evidence from the past that government plans fail quite often in employment trend.
host
 
 

Better Explanation?

by host Sat Jul 12, 2008 11:19 pm

I am not able to see how 'B' weakens the conclusion. Can someone explain a bit more.
RonPurewal
Students
 
Posts: 19744
Joined: Tue Aug 14, 2007 8:23 am
 

Re: CR - The anticipated retirement of tens of thousands...

by RonPurewal Fri Jul 18, 2008 4:47 am

Leon Wrote:1. In agreeing with the answer key that the correct answer is (B), I would have had to assume that "government efforts" referred to the past efforts of the city government in question. Should I? I have seen other CR questions in which this kind of "government efforts" reference implied efforts by governments in general - city, state, or federal government - instead of the specific one in question, and this kind of generalizing answer was wrong because other government efforts are "irrelevant." Am I reading into this answer choice too much, or could this answer choice be phrased better?


you should read government efforts as, well, "government efforts".
in other words, government efforts in general - an umbrella that does, indeed, subsume the efforts of the particular government in question.

also, note the definitive nature of the answer choice: ...have been shown to...
this wording indicates that we have sufficient evidence to KNOW that government's influence on employment trends is very small, if not negligible. since we're talking about government efforts in general, the argument applies to the specific government treated in the problem. if you're going to say that the government in question is somehow exempted from this reasoning, you need to justify that exemption. you DON'T have to justify including government X in a general statement about governments, because, well, government X is a government.


Leon Wrote:2. I have somewhat of a hard time with the explanation for (E): "If anything, this choice strengthens the argument. If these people are unaware of these opportunities, it would be positive to convene to plan how to reach them." When I think of educators, I think of teachers making a difference in the classrooms. These out-of-work young people are presumably out of school, so I did not make a connection between the educators and the out-of-work young people. And, because of the lack of connection, I chose (E), thinking that no matter how hard educators try to come up with the right educational plans, these plans would be meaningless if they are not taught to people. Is there something wrong in my logic?
Leon


this answer choice doesn't provide evidence that the effort will be restricted to classrooms. in fact, the passage talks about convening educators and employers, which is pretty strong evidence that stuff will be happening outside classrooms as well.
in any case, you need some pretty huge assumptions to claim that this choice weakens the argument - assumptions that are much, much, much more extreme than to assume that a statement about governments applies to a government!
guy29
 
 

by guy29 Fri Jan 23, 2009 5:53 pm

Personally, I think the question is a little confusing, for the following reason:

(C) The best available positions require skills not possessed by the vast majority of the unemployed.

Because the word best can modify either positions, as in the best positions which are available, or available, as in those positions which are most easily accessible - C may be and may not be the best answer. Let's assume that best modifies available, in that case most unemployed people will not be able to apply to most jobs, and this directly contradicts one of the assumptions in the argument: "Fortunately, many of these jobs have relatively low barriers to entry and could be filled by out-of-work young people," as the barriers of entry would be significant for most jobs. Though it might be that the ambiguity of the answer would automatically disqualify it, though I don't know if the GMAT would make questions that difficult.
JonathanSchneider
ManhattanGMAT Staff
 
Posts: 477
Joined: Wed Dec 12, 2007 5:40 am
Location: Durham, NC
 

Re: CR - The anticipated retirement of tens of thousands...

by JonathanSchneider Wed Feb 18, 2009 3:27 pm

I see your point, but I disagree. I don't think "best available positions" can mean the positions that are "most easily accessible." We would need to say "the most available positions" in that case. Think about it another way: would you say: "the positions that are best available?" No. This sounds wrong because "best" does not properly modify "available."
vinnieregina
Students
 
Posts: 3
Joined: Wed Aug 12, 2009 3:22 am
 

Re: CR - The anticipated retirement of tens of thousands...

by vinnieregina Wed Sep 23, 2009 4:59 pm

Hello,

I have a question regarding answer choice B. At first glance it seemed like an attractive choice, however I eliminated it because of the use of the words "unprecedented opportunity" in the first sentence.

B suggests the government rarely has the ability to influence employment trends. This makes sense; if they failed in the past, they are likely to fail again under similar circumstances.

However, the situation the government is trying to influence is "unprecedented" - never happened before. Since no situation like this has ever occurred, how can we use past performance to evaluate future performance?

Thanks
RonPurewal
Students
 
Posts: 19744
Joined: Tue Aug 14, 2007 8:23 am
 

Re: CR - The anticipated retirement of tens of thousands...

by RonPurewal Sun Oct 11, 2009 8:32 am

vinnieregina Wrote:Hello,

I have a question regarding answer choice B. At first glance it seemed like an attractive choice, however I eliminated it because of the use of the words "unprecedented opportunity" in the first sentence.

B suggests the government rarely has the ability to influence employment trends. This makes sense; if they failed in the past, they are likely to fail again under similar circumstances.

However, the situation the government is trying to influence is "unprecedented" - never happened before. Since no situation like this has ever occurred, how can we use past performance to evaluate future performance?

Thanks


that's a perceptive point, but you should realize that, while unprecedented, this situation is still categorically related to other employment-related situations.
in other words, while THIS EXACT SITUATION has not occurred before (hence "unprecedented"), other, AT LEAST SOMEWHAT RELATED, situations have occurred. therefore, the past is still a relevant, albeit rough, guideline.

analogy:
let's say that there's an unprecedented heat wave this week, in which temperatures reach 127 degrees Fahrenheit.
even though this heat wave is unprecedented (i.e., the temperatures have never gotten that high before), this clearly doesn't mean that we have to ignore all of our previous experience with heat waves.
kannan_m_80
Course Students
 
Posts: 9
Joined: Mon Mar 02, 2009 4:24 am
 

Re: CR - The anticipated retirement of tens of thousands...

by kannan_m_80 Wed May 18, 2011 11:49 am

Hello all, I am late to this discussion. My question pertains to Strengthen/Weaken questions in general, this question here is a good example to address the doubt I have.

For weaken/Strengthen type questions, do you always work on weakening/strengthening the conclusion. Are we supposed to look for answer choices that will do the same to the conclusion, i.e "To achieve this result, XYZ.." Because answer choice D could work against the premise that is some baby boomers will not retire as expected. Can someone please clarify? thanks.
jnelson0612
ManhattanGMAT Staff
 
Posts: 2664
Joined: Fri Feb 05, 2010 10:57 am
 

Re: CR - The anticipated retirement of tens of thousands...

by jnelson0612 Sun May 22, 2011 9:57 pm

kannan_m_80 Wrote:Hello all, I am late to this discussion. My question pertains to Strengthen/Weaken questions in general, this question here is a good example to address the doubt I have.

For weaken/Strengthen type questions, do you always work on weakening/strengthening the conclusion. Are we supposed to look for answer choices that will do the same to the conclusion, i.e "To achieve this result, XYZ.." Because answer choice D could work against the premise that is some baby boomers will not retire as expected. Can someone please clarify? thanks.


The correct answer absolutely should strengthen or weaken the conclusion; however, the way this occurs can vary. Generally the correct answer will strengthen or weaken an assumption; however, less often the correct answer will strengthen or weaken a premise. Note, however, that since conclusions are supported by premises and assumptions, strengthening or weakening either of these elements of an argument will also affect the conclusion.
Jamie Nelson
ManhattanGMAT Instructor